Need help with easy problem; PrepTest 34, Game 1 Forum

Prepare for the LSAT or discuss it with others in this forum.
Post Reply
UCSD1984

New
Posts: 42
Joined: Mon Jan 10, 2011 8:13 pm

Need help with easy problem; PrepTest 34, Game 1

Post by UCSD1984 » Sat Mar 12, 2011 8:47 pm

Not sure why I'm struggling with a Basic Linear problem. Each time I look at a question, I get overwhelmed by the amount of hypotheticals I have to do in order to answer, which leads me to conclude that I'm missing something from my main diagram. The only thing I can see to diagram on my main diagram is the following:

1.) M and O cannot be in aisle 1
2.) One of the L's must be in either aisle 1 or aisle 9
3.) K is in aisle 2
4.) 2-2-2-2-1 distribution (O being the 1)

As far as rules, I have the following:

1.) JJ cannot
2.) MKM block
3.) K>O>L

Thanks in advance.

User avatar
mickeyD

Bronze
Posts: 357
Joined: Wed Feb 16, 2011 12:43 pm

Re: Need help with easy problem; PrepTest 34, Game 1

Post by mickeyD » Sat Mar 12, 2011 8:54 pm

K > O > L, which means that the entire MKM block must go before O, and at least one L.

Therefore, MKM > O > L. This creates not laws at 8 and 9 for M. Also 7,8,9 for K. Also a not law for O at 9.

The latest you can now possibly place the MKM block is 5,6,7, ending in MKMOL. (Temp 3). The earliest you can place it is immediately after K, at 3,4,5 (Temp 1).

Keep in mind that this allows another L to go before the MKM block, and multiple Ls to go after the MKM block.

Temp 1:
MKM = 3,4,5
-dual option L/J at spots 1 and 9
-some combination of O, L, and J for 6,7,8

L/J K M K M _ _ _ L/J

Temp 2:
MKM = 4,5,6
-dual option L/J at spots 1, 3 and 9
-some combination of O, L, and J for 7,8,9

L/J K L/J M K M _ _ L/J

Temp 3:
MKM = 5,6,7
-this forces O to be in the 8th spot
-this forces L in the 9th spot
-this forces J in the first spot

J K _ _ _ M K M O L
Last edited by mickeyD on Sat Mar 12, 2011 9:09 pm, edited 1 time in total.

UCSD1984

New
Posts: 42
Joined: Mon Jan 10, 2011 8:13 pm

Re: Need help with easy problem; PrepTest 34, Game 1

Post by UCSD1984 » Sat Mar 12, 2011 9:08 pm

mickeyD, thanks for the reply. Are you saying that the rule "Olga's aisle is numbered higher than EITHER of Kurt's aisles" means that Olga's aisle must be numbered higher than BOTH of Kurt's aisles? Perhaps this is what was hurting me. I thought "either" meant "either one of".

User avatar
mickeyD

Bronze
Posts: 357
Joined: Wed Feb 16, 2011 12:43 pm

Re: Need help with easy problem; PrepTest 34, Game 1

Post by mickeyD » Sat Mar 12, 2011 9:10 pm

UCSD1984 wrote:mickeyD, thanks for the reply. Are you saying that the rule "Olga's aisle is numbered higher than EITHER of Kurt's aisles" means that Olga's aisle must be numbered higher than BOTH of Kurt's aisles? Perhaps this is what was hurting me. I thought "either" meant "either one of".
It means both.

UCSD1984

New
Posts: 42
Joined: Mon Jan 10, 2011 8:13 pm

Re: Need help with easy problem; PrepTest 34, Game 1

Post by UCSD1984 » Sat Mar 12, 2011 9:11 pm

Thank you SO much, I really appreciate it. I'm going to start it over now. Thanks again my friend.

Want to continue reading?

Register now to search topics and post comments!

Absolutely FREE!


Post Reply

Return to “LSAT Prep and Discussion Forum”